Proof that a function is continuous

Click For Summary
The discussion focuses on proving the continuity of the function f(x) = x^4 - 7x^3 + 11x^2 + 7x - 12 at the points where f(x) = 0. It identifies that one zero is at x = 1, leading to the factorization of f(x) as (x-1)g(x). The zeros are found to be x = -1, 1, 3, and 4, but the main concern is whether this factorization is sufficient to establish continuity at these points. The participants emphasize that simply finding the zeros does not automatically prove continuity, necessitating further analysis at x = -1, 1, 3, and 4. The conclusion is that additional steps are required to demonstrate continuity at these specific values.
LASmith
Messages
21
Reaction score
0
Prove that the function is continuous when f(x)=0
f(x)=x4-7x3+11x2+7x-12f(c)-\epsilon<f(x)<f(c)+\epsilon

Limits maybe taken, however, we do not have the value for c in the limit equation.
 
Physics news on Phys.org
It looks like you have to solve for x, when f(x) = 0, i.e., find the zeros of f(x).

For one of the zeros, notice that the sum of the coefficients of f(x) is zero. Therefore, f(1) = 0.

So you know that one of the factors of f(x) is (x-1). Use long division or synthetic division to find g(x) such that: f(x) = (x-1)g(x).

Added in Edit.
Notice that: f(-x) = x4+7x3+11x2-7x-12. Therefore, f(-(1)) = 0 .
 
Last edited:
SammyS said:
It looks like you have to solve for x, when f(x) = 0, i.e., find the zeros of f(x).

Solving this gives (x-1)(x+1)(x-4)(x-3)
But is this sufficient to show that it is continuous?
 
LASmith said:
Solving this gives (x-1)(x+1)(x-4)(x-3)
But is this sufficient to show that it is continuous?
Of course not.

The problem is to show that f(x) is continuous when f(x)=0. So the problem has become: show that f(x) is continuous for x = -1, 1, 3, 4 .
 
Question: A clock's minute hand has length 4 and its hour hand has length 3. What is the distance between the tips at the moment when it is increasing most rapidly?(Putnam Exam Question) Answer: Making assumption that both the hands moves at constant angular velocities, the answer is ## \sqrt{7} .## But don't you think this assumption is somewhat doubtful and wrong?

Similar threads

  • · Replies 2 ·
Replies
2
Views
1K
Replies
22
Views
2K
  • · Replies 6 ·
Replies
6
Views
2K
  • · Replies 27 ·
Replies
27
Views
2K
  • · Replies 11 ·
Replies
11
Views
2K
  • · Replies 8 ·
Replies
8
Views
3K
Replies
7
Views
1K
  • · Replies 14 ·
Replies
14
Views
2K
Replies
6
Views
2K
  • · Replies 5 ·
Replies
5
Views
2K